1981 AHSME Problems/Problem 15

Revision as of 19:08, 23 October 2021 by Arcticturn (talk | contribs) (Created page with "==Problem== If <math>b>1</math>, <math>x>0</math>, and <math>(2x)^{\log_b 2}-(3x)^{\log_b 3}=0</math>, then <math>x</math> is <math>\textbf{(A)}\ \dfrac{1}{216}\qquad\textbf...")
(diff) ← Older revision | Latest revision (diff) | Newer revision → (diff)

Problem

If $b>1$, $x>0$, and $(2x)^{\log_b 2}-(3x)^{\log_b 3}=0$, then $x$ is

$\textbf{(A)}\ \dfrac{1}{216}\qquad\textbf{(B)}\ \dfrac{1}{6}\qquad\textbf{(C)}\ 1\qquad\textbf{(D)}\ 6\qquad\textbf{(E)}\ \text{not uniquely determined}$